Difference between revisions of "2002 AMC 10A Problems/Problem 1"

(Solution: fix)
(more scum than alg)
Line 10: Line 10:
 
{{AMC10 box|year=2002|ab=A|before=First question|num-a=2}}
 
{{AMC10 box|year=2002|ab=A|before=First question|num-a=2}}
  
[[Category:Introductory Algebra Problems]]
+
[[Category:Introductory Number Theory Problems]]

Revision as of 13:55, 27 December 2008

Problem

The ratio $\frac{10^{2000}+10^{2002}}{10^{2001}+10^{2001}}$ is closest to which of the following numbers?

$\text{(A)}\ 0.1 \qquad \text{(B)}\ 0.2 \qquad \text{(C)}\ 1 \qquad \text{(D)}\ 5 \qquad \text{(E)}\ 10$

Solution

We factor $\frac{10^{2000}+10^{2002}}{10^{2001}+10^{2001}}$ as $\frac{10^{2000}(1+100)}{10^{2001}(1+1)}=\frac{101}{20}$. As $\frac{101}{20}=5.05$, our answer is $\boxed{\text{(D)}\ 5 }$.

See Also

2002 AMC 10A (ProblemsAnswer KeyResources)
Preceded by
First question
Followed by
Problem 2
1 2 3 4 5 6 7 8 9 10 11 12 13 14 15 16 17 18 19 20 21 22 23 24 25
All AMC 10 Problems and Solutions